Identificarse Registrarse

Psu
Enseñanza Básica
Enseñanza Media
Universidad
Olimpiadas
Comunidad



 
Reply to this topicStart new topic
> Selectivo Ibero 2014
Niklaash
mensaje Aug 6 2014, 10:23 PM
Publicado: #1


Doctor en Matemáticas
Ícono de Grupo

Grupo: Colaborador Silver
Mensajes: 193
Registrado: 17-August 12
Desde: Loncuma :3
Miembro Nº: 110.077
Nacionalidad:
Sexo:



Selectivo equipo chileno
Olimpiada Iberoamericana de Matemáticas 2014

Miércoles, 6 de Agosto 2014


Problema 1: Considere una función TEX: $f: \mathbb{R} \rightarrow \mathbb{R}$ verificando para todo TEX: $x \in \mathbb{R}$

TEX: $\displaystyle f(x+1)=\frac{1}{2} + \sqrt{f(x)-f(x)^2}$

Demuestre que existe TEX: $b>0$ tal que TEX: $f(x+b)=f(x)$ para todo TEX: $x \in \mathbb{R}$


Problema 2: Sea TEX: $\triangle ABC$ un triángulo y puntos TEX: $P,Q,R$ sobre TEX: $\overline{AB},\overline{BC},\overline{AC}$respectivamente, de manera tal que:

TEX: $\displaystyle\frac{AP}{AB}=\frac{BQ}{BC}=\frac{CR}{CA}=\frac{1}{n}$


para TEX: $n \in \mathbb{N}$. Los segmentos TEX: $\overline{AQ}$ y TEX: $\overline{CP}$ se cortan en TEX: $D$, los segmentos TEX: $\overline{BR}$ y TEX: $\overline{AQ}$ se cortan en TEX: $E$ y los segmentos TEX: $\overline{BR}$ y TEX: $\overline{CP}$ se cortan en TEX: $F$. Calcule la razón:

TEX: $\displaystyle\frac{Area (\triangle ABC)}{Area (\triangle DEF)}$


Problema 3: Sea TEX: $x_0=5$ y TEX: $x_{n+1}=x_n+\frac{1}{x_n}$. Demuestre que:

TEX: $\displaystyle 45 < x_{1000} < 45,1$


Tiempo: 2 horas.

Mensaje modificado por Niklaash el Aug 18 2014, 04:50 PM
Go to the top of the page
 
+Quote Post
Adrianocor
mensaje Sep 28 2014, 07:37 PM
Publicado: #2


Dios Matemático
Ícono de Grupo

Grupo: Usuario FMAT
Mensajes: 325
Registrado: 18-March 14
Miembro Nº: 127.725
Nacionalidad:
Colegio/Liceo: Scuola Italiana Vittorio Montiglio
Universidad: Universidad de Chile-FCFM
Sexo:



el problema dos es una aplicación directa del teorema de routh
http://es.wikipedia.org/wiki/Teorema_de_Routh

publican las soluciones oficiales de esto?
Go to the top of the page
 
+Quote Post
vocin
mensaje Sep 28 2014, 07:41 PM
Publicado: #3


Dios Matemático Supremo
Ícono de Grupo

Grupo: Colaborador Silver
Mensajes: 648
Registrado: 26-October 13
Desde: Tokyo-3
Miembro Nº: 123.749
Nacionalidad:
Sexo:



CITA(Adrianocor @ Sep 28 2014, 08:37 PM) *
el problema dos es una aplicación directa del teorema de routh
http://es.wikipedia.org/wiki/Teorema_de_Routh

publican las soluciones oficiales de esto?


Creo que no sad.gif El P2 me contaron que salía aplicando Menelao muchas veces (que por lo demás, es el método que se usa para demostrar el Teorema de Routh aportacion.gif )

EDIT: El P3, cuando me entró la curiosidad y lo mandé a los cracks japochinos de AoPS: http://www.artofproblemsolving.com/Forum/v...36&t=602650

Mensaje modificado por vocin el Sep 28 2014, 07:46 PM


--------------------
Pro Tip: Es siempre recomendable saltarse los posts de Insanee/Legition

I wish, that I could turn back time
'cos now the guilt is all mine
can't live without
the trust from those you love
I know we can't forget the past
you can't forget love & pride
because of that, it's killing me inside

Go to the top of the page
 
+Quote Post
Niklaash
mensaje Sep 28 2014, 08:26 PM
Publicado: #4


Doctor en Matemáticas
Ícono de Grupo

Grupo: Colaborador Silver
Mensajes: 193
Registrado: 17-August 12
Desde: Loncuma :3
Miembro Nº: 110.077
Nacionalidad:
Sexo:



CITA(Adrianocor @ Sep 28 2014, 03:37 PM) *
el problema dos es una aplicación directa del teorema de routh
http://es.wikipedia.org/wiki/Teorema_de_Routh

publican las soluciones oficiales de esto?

Te estan pidiendo demostrar Routh xd (o bien un caso particular) pero dar una solucion asi directa usando el teorema, no esta bien... lo mismo que aca http://www.fmat.cl/index.php?showtopic=80720 te pedian demostrar el teorema de Miquel... y respecto a las soluciones oficiales.. dudo que existan..

Saludos
Go to the top of the page
 
+Quote Post
Adrianocor
mensaje Sep 28 2014, 08:42 PM
Publicado: #5


Dios Matemático
Ícono de Grupo

Grupo: Usuario FMAT
Mensajes: 325
Registrado: 18-March 14
Miembro Nº: 127.725
Nacionalidad:
Colegio/Liceo: Scuola Italiana Vittorio Montiglio
Universidad: Universidad de Chile-FCFM
Sexo:



CITA(Niklaash @ Sep 28 2014, 08:26 PM) *
Te estan pidiendo demostrar Routh xd (o bien un caso particular) pero dar una solucion asi directa usando el teorema, no esta bien... lo mismo que aca http://www.fmat.cl/index.php?showtopic=80720 te pedian demostrar el teorema de Miquel... y respecto a las soluciones oficiales.. dudo que existan..

Saludos


No lo estaba dando como solución, decia nomas, no tiene ninguna lógica ponerlo asi zippytecito.gif
En todo caso todas las demostraciones de routh que me encontre son enormes, debe haber una forma mas corta dado que es un caso particular.

Saludos
Go to the top of the page
 
+Quote Post
mamboraper
mensaje Dec 18 2017, 02:03 PM
Publicado: #6


Maestro Matemático
Ícono de Grupo

Grupo: Usuario FMAT
Mensajes: 134
Registrado: 28-March 14
Miembro Nº: 128.100
Nacionalidad:
Sexo:



P1


--------------------
Hago clases particulares (activo 2024).
Cualquier consulta por MP.
Go to the top of the page
 
+Quote Post
1123581321
mensaje Dec 19 2017, 04:12 PM
Publicado: #7


Matemático
Ícono de Grupo

Grupo: Usuario FMAT
Mensajes: 50
Registrado: 19-December 17
Miembro Nº: 155.290



CITA(Niklaash @ Aug 6 2014, 10:23 PM) *
Problema 3: Sea TEX: $x_0=5$ y TEX: $x_{n+1}=x_n+\frac{1}{x_n}$. Demuestre que:

TEX: $\displaystyle 45 < x_{1000} < 45,1$


Tiempo: 2 horas.


http://www.fmat.cl/index.php?showtopic=17786


--------------------
La intuición es un arma poderosa en un matemático.-

...Un hermoso cálculo que nació en una noche de inspiración
TEX: $$\frac{\sqrt[5]{5}}{\sqrt[3]{3}}\cdot \frac{\sqrt[9]{9}}{\sqrt[7]{7}}\cdot \frac{\sqrt[13]{13}}{\sqrt[11]{11}}\cdot \frac{\sqrt[17]{17}}{\sqrt[15]{15}}\cdot \frac{\sqrt[21]{21}}{\sqrt[19]{19}}\cdot \frac{\sqrt[25]{25}}{\sqrt[23]{23}}\cdot ...=\exp \left( -\frac{\pi \gamma }{4}-\frac{\pi }{2}\log 2-\frac{3\pi }{4}\log \pi +\pi \log \Gamma \left( \frac{1}{4} \right) \right)$$
Go to the top of the page
 
+Quote Post
Milton12
mensaje Sep 3 2018, 10:32 PM
Publicado: #8


Principiante Matemático
Ícono de Grupo

Grupo: Usuario FMAT
Mensajes: 1
Registrado: 28-August 18
Miembro Nº: 158.904



El P1 es de la IMO del 68
Go to the top of the page
 
+Quote Post

Reply to this topicStart new topic
1 usuario(s) está(n) leyendo esta discusión (1 invitado(s) y 0 usuario(s) anónimo(s))
0 miembro(s):

 

Versión Lo-Fi Fecha y Hora actual: 23rd November 2024 - 05:54 PM